Jolie intégrale

Bonjour
Montrer que $$\int_0^1 \left(\arctan x \right)^3\,dx \qquad=\quad
\frac1{64} \big(-48 \pi C + 63 \zeta(3)+ \pi^2 (\pi+ \log(64))\big).

$$ https://www.wolframalpha.com/input/?i=\int_0^1+\left(\arctan+x+\right)^3\,dx
Le 😄 Farceur


Réponses

  • Par IPP successivese
  • Qui est $C$ ?
  • C'est $G$, la constante de Catalan.

    PS:
    Je suis curieux de voir les IPP successives indiquées par Fjaclot, que je salue.
  • FDP avec ta méthode (dans le cas d'un carré) ça marche encore pour un cube?
    Le 😄 Farceur


  • Gebrane:
    Bof.

    Cela ne répond pas à la question mais voici comment je calculais $\displaystyle \int_0^1 \arctan^2 x\,dx$ en 2014:

    Si on sait que $\displaystyle \int_0^{\dfrac{\pi}{2}}\log (\sin (x))dx=-\dfrac{\pi}{2}\log(2)$

    Alors :

    $\displaystyle \int_0^\dfrac{\pi}{2}\dfrac{x}{\tan x }dx=\dfrac{\pi}{2}\log(2)$ (IPP)

    Soit $I=\displaystyle \int_0^\dfrac{\pi}{2}\dfrac{x}{\tan x }dx$

    Appliquer le changement de variable $u=\tan x$ :


    $I=\displaystyle \int_0^{+\infty} \dfrac{\arctan x}{x(1+x^2)}dx$

    $I=\displaystyle \int_0^1 \dfrac{\arctan x}{x(1+x^2)}dx+\int_1^{+\infty} \dfrac{\arctan x}{x(1+x^2)}dx$

    Dans la seconde intégrale dans le membre de droite faire le changement de variable $u=\dfrac{1}{x }$
    $I= \displaystyle\int_0^1 \dfrac{\arctan x}{x(1+x^2)}dx+ \int_0^1 \dfrac{x\arctan
    \Big(\dfrac{1}{x}\Big) }{1+x^2}dx$

    Pour $x>0$ on a $\arctan\Big(\dfrac{1}{x}\Big)+\arctan x=\dfrac{\pi}{2}$


    $I= \displaystyle\int_0^1 \dfrac{\arctan x}{x(1+x^2)}dx+\dfrac{\pi}{2}\int_0^1\dfrac{x}{1+x^2}dx-\int_0^1\dfrac{x\arctan x}{1+x^2}dx$

    Mais:
    $\dfrac{1}{x(1+x^2)}=\dfrac{1}{x}-\dfrac{x}{1+x^2}$

    donc:

    $I=\displaystyle\int_0^1 \dfrac{\arctan x}{x}dx-\displaystyle\int_0^1 \dfrac{x\arctan x}{1+x^2}dx+\dfrac{\pi}{2}\int_0^1\dfrac{x}{1+x^2}dx-\int_0^1\dfrac{x\arctan x}{1+x^2}dx$

    D'où:
    $I=\displaystyle\int_0^1 \dfrac{\arctan x}{x}dx-2\displaystyle\int_0^1 \dfrac{x\arctan x}{1+x^2}dx+\dfrac{\pi}{4}\Big[\log(1+x^2)\Big]_0^1$

    et on a aussi:

    $I=\displaystyle\int_0^1 \dfrac{\arctan x}{x}dx-2\displaystyle\int_0^1 \dfrac{x\arctan x
    }{1+x^2}dx+\dfrac{\pi}{4}\log(2)$

    Mais:
    la fonction dérivée de $x\rightarrow (\arctan x)^2$ est $x\rightarrow \dfrac{2\arctan x }{1+x^2}$

    donc:

    $I=\displaystyle\int_0^1 \dfrac{\arctan x}{x}dx-\Big(\big[ x(\arctan x)^2\big]_0^1-\int_0^1(\arctan x)^2 dx\Big)+\dfrac{\pi}{4}\log(2)$
    $I=\displaystyle\int_0^1\dfrac{\arctan x }{x}dx-\dfrac{\pi^2}{16}+\int_0^1(\arctan x)^2 dx$

    mais $I=\dfrac{\pi}{2}\log(2)$ ainsi:

    $\displaystyle\int_0^1 (\arctan x)^2
    dx=\dfrac{\pi^2}{16}-G+\dfrac{\pi}{4}\log(2)$

    et $\displaystyle G=\int_0^1 \dfrac{\arctan x}{x}dx$ est la constante de Catalan.
  • Salut bien cordial en retour a FDP

    1/ x= tan(t)
    2/ 1ere IPP
    3/ 2e IPP
    4/ serie de Fourier de Ln(cos(t))

    Y-a-t-il une solution pour I(4) ?
    Et pour I(1/2) ?

    fjaclot
  • FDP, je faisais allusion à cette méthode , mais dans le cas d'un cube, le bon départ serait à partir de $I=$ https://www.wolframalpha.com/input/?i=\int_0^{\pi+/2}++{x^2/+tan+x+}+ ?
    Le 😄 Farceur


  • Gebrane:

    C'est grosso-modo ce que propose Fjaclot si j'ai bien suivi.

    Une primitive de $x\rightarrow \dfrac{1}{\tan x}$ est $x\rightarrow \ln(\sin x)$.
    On connait le développement en série de Fourier de cette dernière fonction.

    Je pense que c'est de cette façon que Wolfy calcule ces intégrales.
  • J'ai peut-être une solution alternative au calcul demandé dans le premier message qui n'utilise pas de série de Fourier. Cette solution reposerait sur le calcul de l'intégrale $\displaystyle \int_0^1 \frac{\ln(1+x^2)\arctan x}{1+x^2}\,dx$, entre autres.

    En effet,

    \begin{align}J&=\int_0^1 \arctan^3 x\,dx\\
    &\overset{\text{IPP}}=\Big[x\arctan^3 x\Big]_0^1 -3\int_0^1 \frac{x\arctan^2 x}{1+x^2}\,dx\\
    &=\frac{\pi^3 }{64}-3\int_0^1 \frac{x\arctan^2 x}{1+x^2}\,dx\\
    &\overset{\text{IPP}}=\frac{\pi^3 }{64}-\frac{3}{2}\left[\ln(1+x^2)\arctan^2 x\right]_0^1 +3\int_0^1 \frac{\ln(1+x^2)\arctan x}{1+x^2}\,dx\\
    &=\frac{\pi^3 }{64}-\frac{3\pi^2\ln 2}{32}+3\int_0^1 \frac{\ln(1+x^2)\arctan x}{1+x^2}\,dx\\
    \end{align}
    Il reste le plus difficile à faire mais je suis confiant dans la faisabilité. ;-)
  • Bonsoir FDP
    on verra le plus rapide moi ou toi ou eux :-D
    Si tu trouves le premier , envoie moi le lien de ton fichier sans le mot de passe :-P
    Le 😄 Farceur


  • Ils ont trouvé pendant mon sommeil :-(
    Le 😄 Farceur


  • Gebrane: qui sont ce ILS?

    Je n'ai pas bien regardé sur M.E peut-être.
  • C'est sur Me . Continu ton calcul
    Le 😄 Farceur


  • \begin{align}
    J&=\int_0^1 \arctan^3 x\,dx\\
    &\overset{\text{IPP}}=\Big[x\arctan^3 x\Big]_0^1 -3\int_0^1 \frac{x\arctan^2 x}{1+x^2}\,dx\\
    &=\frac{\pi^3 }{64}-3\int_0^1 \frac{x\arctan^2 x}{1+x^2}\,dx\\
    &\overset{\text{IPP}}=\frac{\pi^3 }{64}-\frac{3}{2}\left[\ln(1+x^2)\arctan^2 x\right]_0^1 +3\int_0^1 \frac{\ln(1+x^2)\arctan x}{1+x^2}\,dx\\
    &=\frac{\pi^3 }{64}-\frac{3\pi^2\ln 2}{32}+3\int_0^1 \frac{\ln(1+x^2)\arctan x}{1+x^2}\,dx\\
    &\overset{x=\tan t}=\frac{\pi^3 }{64}-\frac{3\pi^2\ln 2}{32}-6\int_0^{\frac{\pi}{4}}t\ln(\cos t)\,dt\\
    A&=\int_0^{\frac{\pi}{4}}t\ln(\cos t)\,dt\\
    B&=\int_0^{\frac{\pi}{4}}t\ln(\sin t)\,dt\\
    B-A&=\int_0^{\frac{\pi}{4}}t\ln(\tan t)\,dt\\
    &\overset{x=\tan t}=\int_0^1 \frac{\arctan x\ln x}{1+x^2}\,dx\\
    \end{align}
    On définit sur $[0;\infty]$ la fonction $R$ par,
    pour tout $x\in [0;\infty]$, $\displaystyle \text{R}(x)=\int_0^x \dfrac{\ln t}{1+t^2}\,dt=\int_0^1 \dfrac{x\ln(tx)}{1+t^2x^2}\,dt$.
    Remarquer que $\text{R}(0)=\text{R}(\infty)=0$ et $\text{R}(1)=-\text{G}$
    \begin{align}
    U&=\int_0^1 \frac{\arctan\left(\frac{1}{x}\right)\ln x}{1+x^2}\,dx\\
    V&=\int_0^1 \frac{\arctan x\ln x}{1+x^2}\,dx\\
    U+V&=-\frac{1}{2}\pi\text{G}\\
    U&\overset{\text{IPP}}=\left[R(x)\arctan\left(\frac{1}{x}\right)\right]_0^1 +\int_0^1 \frac{R(x)}{1+x^2}\,dx\\
    &=-\frac{1}{4}\pi\text{G}+\int_0^1 \int_0^1 \frac{x\ln(tx)}{(1+x^2)(1+t^2x^2)}\,dt\,dx\\
    &=-\frac{1}{4}\pi\text{G}+\int_0^1 \left(\int_0^1 \dfrac{x\ln(x)}{(1+t^2x^2)(1+x^2)}\,dt\right)\,dx+\int_0^1 \left(\int_0^1 \dfrac{x\ln(t)}{(1+t^2x^2)(1+x^2)}\,dx\right)\,dt\\
    &=-\frac{1}{4}\pi\text{G}+V+\frac{1}{2}\ln 2\int_0^1 \frac{\ln t}{1-t^2}\,dt-\frac{1}{2}\int_0^1 \frac{\ln t\ln(1+t^2)}{1-t^2}\,dt\\
    &=-\frac{1}{4}\pi\text{G}+V-\frac{1}{16}\pi^2\ln 2-\frac{1}{2}\int_0^1 \frac{\ln t\ln(1+t^2)}{1-t^2}\,dt\\
    Z&=\int_0^1 \frac{\ln x\ln(1+x^2)}{1-x^2}\,dx
    \end{align}
    On définit sur $[0;1]$ la fonction $S$ par,
    pour tout $x\in [0;1]$, $\displaystyle S(x)=\int_0^x \frac{\ln t}{1-t^2}\,dt=\int_0^1 \frac{x\ln (tx)}{1-t^2x^2}\,dt$
    Remarquer que $\displaystyle S(0)=0,S(1)=-\frac{1}{8}\pi^2$.
    \begin{align}Z&\overset{\text{IPP}}=\Big[S(x)\ln(1+x^2)\Big]_0^1-\int_0^1 \frac{2xS(x)}{1+x^2}\,dx\\
    &=-\frac{1}{8}\pi^2\ln 2-\int_0^1 \int_0^1 \frac{2x^2\ln(tx)}{(1+x^2)(1-t^2x^2)}\,dt\,dx\\
    &=-\frac{1}{8}\pi^2\ln 2-\int_0^1 \left(\int_0^1 \frac{2x^2\ln x}{(1+x^2)(1-t^2x^2)}\,dt\right)\,dx-\int_0^1 \left(\int_0^1 \frac{2x^2\ln t}{(1+x^2)(1-t^2x^2)}\,dx\right)\,dt\\
    &=-\frac{1}{8}\pi^2\ln 2+\int_0^1 \frac{x\ln\left(\frac{1-x}{1+x}\right)\ln x}{1+x^2}\,dx-\\
    &\left(\int_0^1 \frac{\ln(1+t)\ln t}{t}\,dt+\int_0^1 \frac{t\ln\left(\frac{1-t}{1+t}\right)\ln t}{1+t^2}\,dt-\int_0^1 \frac{\ln(1-t)\ln t}{t}\,dt-\frac{\pi}{2}\int_0^1 \frac{\ln t}{1+t^2}\,dt\right)\\
    &=-\frac{1}{8}\pi^2\ln 2-\int_0^1 \frac{\ln(1+t)\ln t}{t}\,dt+\int_0^1 \frac{\ln(1-t)\ln t}{t}\,dt-\frac{1}{2}\pi\text{G}\\
    &=-\frac{1}{8}\pi^2\ln 2+\frac{7}{4}\zeta(3)-\frac{1}{2}\pi\text{G}\\
    \end{align}
    Ainsi,
    \begin{align}U&=V-\frac{7}{8}\zeta(3)\\
    V&=\frac{7}{16}\zeta(3)-\frac{1}{4}\pi\text{G}\\
    B-A&=\frac{7}{16}\zeta(3)-\frac{1}{4}\pi\text{G}\\
    U&=-\frac{7}{16}\zeta(3)-\frac{1}{4}\pi\text{G}\\
    \end{align}
    Par ailleurs,
    \begin{align}
    A+B&=\int_0^{\frac{\pi}{4}}t\ln\left(\frac{1}{2}\sin(2t)\right)\,dt\\
    &=\int_0^{\frac{\pi}{4}}t\ln\left(\sin(2t)\right)\,dt-\frac{\pi^2\ln 2}{32}\\
    &\overset{x=2t}=\frac{1}{4}\int_0^{\frac{\pi}{2}}x\ln\left(\sin x\right)\,dx-\frac{\pi^2\ln 2}{32}\\
    A_2&=\int_0^{\frac{\pi}{2}}t\ln(\cos t)\,dt\\
    B_2&=\int_0^{\frac{\pi}{2}}t\ln(\sin t)\,dt\\
    A_2+B_2&=\int_0^{\frac{\pi}{2}}t\ln\left(\frac{1}{2}\sin(2t)\right)\,dt\\
    &=\int_0^{\frac{\pi}{2}}t\ln\left(\sin(2t)\right)\,dt-\frac{\pi^2\ln 2}{8}\\
    &\overset{x=2t}=\frac{1}{4}\int_0^\pi x\ln(\sin x)\,dx-\frac{\pi^2\ln 2}{8}\\
    &\overset{t=\pi-x}=\frac{1}{4}\int_0^\pi (\pi-x)\ln(\sin x)\,dx-\frac{\pi^2\ln 2}{8}\\
    2(A_2+B_2)&=\frac{\pi}{4}\int_0^\pi \ln(\sin x)\,dx-\frac{\pi^2\ln 2}{4}\\
    A_2+B_2&=\frac{\pi}{8}\int_0^\pi \ln(\sin x)\,dx-\frac{\pi^2\ln 2}{8}\\
    &=-\frac{\pi^2\ln 2}{4}\\
    B2-A2&=\int_0^{\frac{\pi}{2}}t\ln(\tan t)\,dt\\
    &\overset{x=\tan t}=\int_0^\infty \frac{\ln x\arctan x}{1+x^2}\,dx\\
    \end{align}
    \begin{align}
    U_2&=\int_0^\infty \frac{\arctan\left(\frac{1}{x}\right)\ln x}{1+x^2}\,dx\\
    V_2&=\int_0^\infty \frac{\arctan\left(x\right)\ln x}{1+x^2}\,dx\\
    U_2+V_2&=\frac{\pi}{2}\int_0^\infty \frac{\ln x}{1+x^2}\,dx\\
    &=0\\
    U_2&\overset{\text{IPP}}=\left[R(x)\arctan\left(\frac{1}{x}\right)\right]_0^\infty +\int_0^\infty \frac{R(x)}{1+x^2}\,dx\\
    &=\int_0^\infty \left(\int_0^1 \dfrac{x\ln(tx)}{(1+t^2x^2)(1+x^2)}\,dt\right)\,dx\\
    &=\int_0^\infty \left(\int_0^1 \dfrac{x\ln(x)}{(1+t^2x^2)(1+x^2)}\,dt\right)\,dx+\int_0^1 \left(\int_0^\infty \dfrac{x\ln(t)}{(1+t^2x^2)(1+x^2)}\,dx\right)\,dt\\
    &=V_2+\int_0^1 \frac{\ln^2 t}{t^2-1}\,dt\\
    &=V_2+\int_0^1 \frac{t\ln^2 t}{1-t^2}\,dt-\int_0^1 \frac{\ln^2 x}{1-x}\,dx\\
    &\overset{u=t^2}=B+\frac{1}{8}\int_0^1 \frac{\ln^2 t}{1-t}\,dt-\int_0^1 \frac{\ln^2 x}{1-x}\,dx\\
    &=V_2-\frac{7}{8}\int_0^1 \frac{\ln^2 x}{1-x}\,dx\\
    &=V_2-\frac{7}{8}\times 2\zeta(3)\\
    &=V_2-\frac{7}{4}\zeta(3)\\
    U_2&=-\frac{7}{8}\zeta(3)\\
    V_2&=\frac{7}{8}\zeta(3)\\
    B_2-A_2&=\frac{7}{8}\zeta(3)\\
    A_2&=-\frac{7}{16}\zeta(3)-\frac{1}{8}\pi^2\ln 2\\
    B_2&=\frac{7}{16}\zeta(3)-\frac{1}{8}\pi^2\ln 2\\
    A+B&=\frac{7}{64}\zeta(3)-\frac{1}{16}\pi^2\ln 2\\
    A&=\frac{1}{8}\pi\text{G}-\frac{21}{128}\zeta(3)-\frac{1}{32}\pi^2\ln 2\\
    B&=\frac{35}{128}\zeta(3)-\frac{1}{32}\pi^2\ln 2-\frac{1}{8}\pi\text{G}\\
    J&=\frac{\pi^3 }{64}-\frac{3\pi^2\ln 2}{32}-6A\\
    J&=\boxed{\dfrac{1}{64}\pi^3+\dfrac{3}{32}\pi^2\ln 2-\dfrac{3}{4}\pi\text{G}+\dfrac{63}{64}\zeta(3)}
    \end{align}

    NB: J'ai utilisé les résultats suivants:
    \begin{align}x>0,\arctan(x)+\arctan\left(\frac{1}{x}\right)&=\frac{\pi}{2}\\
    \int_0^\infty \frac{\ln x}{1+x^2}\,dx&=0\\
    \int_0^{\frac{\pi}{2}}\ln(\cos x)\,dx&=-\frac{1}{2}\pi\ln 2\\
    \int_0^1 \frac{\ln x}{1-x^2}\,dx=-\frac{1}{8}\pi^2\\
    \int_0^1 \frac{\ln^2 x}{1-x}\,dx&=2\zeta(3)\\
    \int_0^1 \frac{\ln(1+x)\ln x}{x}\,dx&=-\frac{3}{4}\zeta(3)\\
    \int_0^1 \frac{\ln(1-x)\ln x}{x}\,dx&=\zeta(3)\\
    \int_0^\pi \ln(\sin x)\,dx&=-\pi\ln 2
    \end{align}
  • Incroyable !
    J'ai bien fait d' attendre d 'accepter une réponse.
    Le 😄 Farceur


  • Dans un autre fil http://www.les-mathematiques.net/phorum/read.php?4,1481230,1493856#msg-1493856 j'avais déjà fait une partie des calculs ci-dessus (j'avais un sentiment de déjà-vu). Il était question de calculer:
    $\displaystyle K=\int_0^{\frac{\pi}{2}} \frac{x^2}{\sin x}dx$
Connectez-vous ou Inscrivez-vous pour répondre.